Đến nội dung

Hình ảnh

Topic luyện thi vào lớp 10 năm 2013 – 2014 (Hình học)


  • Please log in to reply
Chủ đề này có 748 trả lời

#261
CaptainAmerica

CaptainAmerica

    Hạ sĩ

  • Thành viên
  • 64 Bài viết

Bài 95 :
Cho đường tròn (O;R) đường kính AB. Qua B kẻ tiếp tuyến d của đường tròn (O). MN là một đường kính thay đổi của đường tròn ( M không trùng với A,B ). Các đường thẳng AM và AN cắt đường thẳng d lần lượt tại C và D.
a) Cm : AM.AC = AN.AD
b) Tìm giá trị nhỏ nhất của tích AC.AD
c) Cm tâm đường tròn ngoại tiếp ∆MNC thuộc một đường thẳng cố định.
d) Gọi I là giao điểm của CO và BM. Đường thẳng AI cắt đường tròn (O) tại điểm thứ hai là E, cắt đường thẳng d tại F. Cm : 3 điểm C, E, N thẳng hàng.

Mình sẽ nói hướng làm thôi chứ làm biến ghi lời giải lắm... ^^
a) Ta sẽ CM được: $\Delta AMN \sim \Delta ADC$ ( 1 góc vuông chung, $do \widehat{OAN}=\widehat{ONA} (\Delta cân) \Rightarrow \widehat{AMN}=\widehat{ADC})$
Từ đó $\Rightarrow$ đpcm
b) Ta có: AC.AD=AB.CD(HTL)
Mà AB=2R không đổi
$\Rightarrow AC.AD$ phụ thuộc vào CD.
Lại có: CD $\geq MN$
Đẳng thức xảy ra $\Leftrightarrow MN là đtb \Delta ACD$
$\Rightarrow CD = 4R$
$\Rightarrow AC.AD=AB.CD=2R.4R=8R^{2}$ ( Làm đại nhe >"< )
c) Hình như là nằm trên pg ngoài của $\widehat{ABC}$ (dự đoán thôi :">)
d) Chắc là cm bằng góc đối đỉnh
Nhưng chưa nghĩ ra... đói quá... thôi nhường cho bạn khác giải vậy :">

Y so serious?


#262
henry0905

henry0905

    Trung úy

  • Thành viên
  • 892 Bài viết

Mình sẽ nói hướng làm thôi chứ làm biến ghi lời giải lắm... ^^
a) Ta sẽ CM được: $\Delta AMN \sim \Delta ADC$ ( 1 góc vuông chung, $do \widehat{OAN}=\widehat{ONA} (\Delta cân) \Rightarrow \widehat{AMN}=\widehat{ADC})$
Từ đó $\Rightarrow$ đpcm
b) Ta có: AC.AD=AB.CD(HTL)
Mà AB=2R không đổi
$\Rightarrow AC.AD$ phụ thuộc vào CD.
Lại có: CD $\geq MN$
Đẳng thức xảy ra $\Leftrightarrow MN là đtb \Delta ACD$
$\Rightarrow CD = 4R$
$\Rightarrow AC.AD=AB.CD=2R.4R=8R^{2}$ ( Làm đại nhe >"< )
c) Hình như là nằm trên pg ngoài của $\widehat{ABC}$ (dự đoán thôi :">)
d) Chắc là cm bằng góc đối đỉnh
Nhưng chưa nghĩ ra... đói quá... thôi nhường cho bạn khác giải vậy :">

Bạn có CD$\geq$MN thì đẳng thức xảy ra là CD=MN (không thể), sao bạn suy ra được 2MN=CD??
C/m:
CD=CB+BD $\geq$ $2\sqrt{AB^{2}}$=4R
Dấu = xảy ra CB=BD=2R. Phần còn lại thì giống bạn captain

Bài viết đã được chỉnh sửa nội dung bởi henry0905: 01-06-2012 - 12:14


#263
davildark

davildark

    Thượng sĩ

  • Thành viên
  • 223 Bài viết

Bài 98: Cho tam giác ABC nội tiếp (O). M thuộc cung BC không chứa A. Hạ MH vuông góc BC; ME vuông góc AB; MF vuông góc AC. Lấy B' đối xứng M qua E; C' đối xứng M qua F; A' đối xứng M qua H.
a, CM: E, H, F thẳng hàng
b, CM: B', A', C' thẳng hàng
c, Gọi V là trực tâm của tam giác ABC.
CM: tứ giác AVBB', AVCC' nội tiếp. Từ đó c/m B'C' đi qua V.
d, Gọi I là giao của VM và EF. CM: I là trung điểm của VM

Đề sai rồi
Đề đúng phải là lấy đối xứng qua các đoạn AB BC AC chứ không phải là các điểm E ,H , F
Chém lun :D
98.JPG
a) Đường thẳng Simson
b) Sử dụng đường trung bình
c) $\widehat{KVB}=\widehat{ACB}=\widehat{AMB}=\widehat{AB'B}$
$\Rightarrow$ tứ giác AVBB' nội tiếp tương tự tứ giác cũng AVCC' nội tiếp
$\Rightarrow \widehat{B'VB}+\widehat{C'VC}=\widehat{B'AB}+\widehat{C'AC}=\widehat{BAM}+\widehat{CAM}=\widehat{BAC}$
Mà $\widehat{BVC}+\widehat{BAC}=180^{\circ}\Rightarrow \widehat{BVC}+\widehat{B'VB}+\widehat{C'VC}=180^{\circ}$
Từ đó => B'C' đi qua V.
d) Sử dụng đường trung bình

Bài viết đã được chỉnh sửa nội dung bởi davildark: 01-06-2012 - 12:22


#264
Doilandan

Doilandan

    Trung sĩ

  • Thành viên
  • 141 Bài viết
Bài 98 có nội dung tương tự bài 92. Vẫn là đường thẳng Stai-nơ.

................................................................

Các bạn giúp mình bài 95.
HÌNH bài 95 :
Hình đã gửi

Bài viết đã được chỉnh sửa nội dung bởi Doilandan: 01-06-2012 - 15:57


#265
hoclamtoan

hoclamtoan

    Thượng sĩ

  • Thành viên
  • 274 Bài viết

Bài 98 có nội dung tương tự bài 92. Vẫn là đường thẳng Stai-nơ.

................................................................

Các bạn giúp mình bài 95.
HÌNH bài 95 :
Hình đã gửi

c) Gọi O' là tâm của đt (MNC) và cũng là tâm của đt (MNDC) nên O' là giao các đường trung trực của MN, DC và từ đó cm được AJO'O là hbh (J là trung điểm của DC) suy ra JO' = OA = R không đổi.
Vậy O' thuộc đường song song với d cố địng và cách d một khoảng không đổi R.
d) $\widehat{MEA}=\widehat{MNA}=\widehat{ACD}\Rightarrow MEFC$ nội tiếp.
$\Rightarrow \widehat{MCE}=\widehat{MFA}=\widehat{MDA}=\widehat{MCN}$
$\Rightarrow CE\equiv CN\Rightarrow$ đpcm.

#266
Doilandan

Doilandan

    Trung sĩ

  • Thành viên
  • 141 Bài viết
HOAN HÔ ! :lol: :namtay :lol:
.........................................................
Topic của chúng ta đã không còn những bài "cù cặn".
Xin mời các bạn tiếp tục post bài mới!

Bài viết đã được chỉnh sửa nội dung bởi Doilandan: 01-06-2012 - 17:30


#267
haku139

haku139

    Binh nhì

  • Thành viên
  • 12 Bài viết
Bài 99: Cho tam giác ABC. M là một điểm thay đổi trên BC. Gọi D và E lần lượt là điểm đối xứng của M qua AB và AC. CMR: trung điểm của DE thuộc một đường thẳng cố định.

Bài 100: Cho tam giác ABC. Đường tròn (I) nội tiếp tam giác ABC và tiếp xúc với BC, AC và AB lần lượt tại D,E,F. AD cắt (I) tại điểm thứ 2 là M. BM và CM cắt (I) tại Y và Z. CMR BZ, CY và AD đồng quy.

Bài viết đã được chỉnh sửa nội dung bởi haku139: 01-06-2012 - 19:25


#268
Math Is Love

Math Is Love

    $\mathfrak{Forever}\ \mathfrak{Love}$

  • Thành viên
  • 620 Bài viết

Bài 99: Cho tam giác ABC. M là một điểm thay đổi trên AB và AC. Gọi D và E lần lượt là điểm đối xứng của M qua AB và AC. CMR: trung điểm của DE thuộc một đường thẳng cố định.

Bạn xem lại đề bài 99.Đề bài không chặt

Hình đã gửi


#269
haku139

haku139

    Binh nhì

  • Thành viên
  • 12 Bài viết

Bạn xem lại đề bài 99.Đề bài không chặt

Đã chỉnh sửa đề bài. Sorry :)

#270
Doilandan

Doilandan

    Trung sĩ

  • Thành viên
  • 141 Bài viết
Bài 99 và 100 đặt ở topic này không thích hợp.
Đây là nơi dành cho các bạn thi không chuyên.
Xin bạn haku139 rút kinh nghiệm.

#271
tolaphuy10a1lhp

tolaphuy10a1lhp

    Thượng sĩ

  • Thành viên
  • 224 Bài viết
Bài 101:

Cho $\triangle ABC(AB<AC)$ có ba góc nhọn nội tiếp $(O)$ . Tiếp tuyến tại A cắt BC tại M. Gọi K là trung điểm BC.

a) Chứng minh tứ giác MAOK nội tiếp.
b) Vẽ đường cao BD và CE của $\triangle ABC$. Chứng minh $ED||MA.$
c) Khi B là trung điểm MC. Chứng minh $AE=AD\sqrt{2}$
d) Gọi H là trung điểm của ED. Tia AH cắt $(O)$ tại L . Chứng minh ML là tiếp tuyến của $(O)$.
Học là ..... hỏi ...............

#272
davildark

davildark

    Thượng sĩ

  • Thành viên
  • 223 Bài viết

Bài 99: Cho tam giác ABC. M là một điểm thay đổi trên BC. Gọi D và E lần lượt là điểm đối xứng của M qua AB và AC. CMR: trung điểm của DE thuộc một đường thẳng cố định.

Bài 100: Cho tam giác ABC. Đường tròn (I) nội tiếp tam giác ABC và tiếp xúc với BC, AC và AB lần lượt tại D,E,F. AD cắt (I) tại điểm thứ 2 là M. BM và CM cắt (I) tại Y và Z. CMR BZ, CY và AD đồng quy.

Chém 2 bài này lun nghĩ mãi mới ra :D
Bài 99 ( Hình như sai ) :D
99.JPG
Gọi I là trung điểm DE
Vẽ các đường cao BF và CH
Dể dàng CM IJMK là hình bình hành ( đường trung bình)
Áp dụng Thales ta có
$\frac{JI}{BF}=\frac{MK}{BF}=\frac{MC}{BC}=\frac{HJ}{HB}$ mà $\widehat{HJI}=\widehat{HBF}$
$\Rightarrow \bigtriangleup HJI\sim \bigtriangleup HBF\Rightarrow \widehat{JHI}=\widehat{BHF}$
$\Rightarrow$ H , I ,F thẳng hàng
Mà H , F cố định vậy I thuộc đoạn HF cố định

Bài 100
Bài này mình sử dụng định lý Ceva và tham khảo các định lí về tứ giác điều hòa các bạn có thể dễ dàng tìm nó tren google
100.JPG
Quay trở lại bài toán
Tứ giác MEZD là tứ giác điều hòa nên $ED.MZ=2EZ.MD\Rightarrow ED.MZ.MC=2EZ.MD.MC=2MD.ME.CE$ (1)
(VÌ $\bigtriangleup CEZ\sim \bigtriangleup CME\Rightarrow EZ.MC=ME.CE$)
Mà EC là tiếp tuyến => $MZ.MC=EC^2$(2)
Chia (1) cho (2) $\Rightarrow \frac{ZC}{MZ.DE}=\frac{EC}{2MD.ME}\Rightarrow \frac{ZC}{MZ}=\frac{EC.DE}{2MD.ME}$
$\Rightarrow \frac{BD}{DC}.\frac{ZC}{MC}=\frac{BF}{EC}.\frac{EC.DE}{2MD.ME}=\frac{BF.DE}{2MD.ME}$
Làm tương tự như trên ta có $\frac{MY}{YB}=\frac{2MD.MF}{BF.FD}$
Kết hợp với trên ta có
$\frac{BD}{DC}.\frac{ZC}{MZ}.\frac{MY}{YB}=\frac{DE.FB}{2MD.ME}.\frac{2MD.MF}{BF.FD}=\frac{FB.MF}{ME.FD}$
Mà DFME là tứ giác điều hòa nên $ FB.MF=ME.FD$
$\Rightarrow \frac{BD}{DC}.\frac{ZC}{MZ}.\frac{MY}{YB}=1$
Theo định lí Ceva ta có BZ, CY và AD đồng quy.

P/s vi đây là đề thi chuyên nên khá là khó hiểu có gì các bạn cứ hỏi

#273
ga nhep

ga nhep

    Binh nhì

  • Thành viên
  • 19 Bài viết
Bài 102:
Cho hình thang ABCD (AB//CD). I, T lần lượt là trung điểm của AB, DC. Trên tia đối của tia AC lấy điểm K bất kì. KI cắt BC tại N, KT cắt AD tại M. Chứng minh: MN // DC

#274
quangdung1997

quangdung1997

    Binh nhất

  • Thành viên
  • 36 Bài viết
Bài 103:Từ điểm A nằm ngoài đường tròn (O),vẽ 2 tiếp tuyến AE,AF và cát tuyến AID,OK vuông góc ID tại K.Chứng minh
a)A,E,O,K,F cùng thuộc 1 đường tròn
b)AI+AD=2AK và $\angle IFE= \angle KFD$
c)FI.DE+FD.IE=FE.ID
d)Tiếp tuyến tại D của đường tròn (O) lần lượt cắt AF và AE tại B,C.Vẽ DH vuông góc EF tại H.Đường trung trực BC cắt EF tại M.Chứng minh BHMC nội tiếp

SỐNG YÊN VUI DANH LỢI MÃI COI THƯỜNG

TÂM BẤT BIẾN GIỮA DÒNG ĐỜI VẠN BIẾN


#275
Doilandan

Doilandan

    Trung sĩ

  • Thành viên
  • 141 Bài viết
BÀI 102 :
Hình đã gửi
Gọi :
$\begin{array}{l}
KN \cap DC = F;\,\,BA \cap KT = J\\
\left\{ \begin{array}{l}
\frac{{JA}}{{TC}} = \frac{{JA}}{{DT}} = \frac{{KA}}{{KC}}(1)\\
\frac{{AI}}{{CF}} = \frac{{IB}}{{CF}} = \frac{{KA}}{{KC}}(2)
\end{array} \right.
\end{array}$.
Từ (1);(2) $ \Rightarrow \frac{{JA}}{{DT}} = \frac{{IB}}{{CF}} \Rightarrow \frac{{AM}}{{MD}} = \frac{{BN}}{{NC}}$.
Mà AB // DC nên MN // DC.

Bài viết đã được chỉnh sửa nội dung bởi Doilandan: 03-06-2012 - 23:33


#276
Doilandan

Doilandan

    Trung sĩ

  • Thành viên
  • 141 Bài viết
Hình bài 103:Hình đã gửi

#277
thusang3605

thusang3605

    Binh nhì

  • Thành viên
  • 17 Bài viết
Bài 104: Cho tam giác ABC vuông tại A. Một đường tròn (O) đi qua B và C cắt các cạnh AB, AC của tam giác ABC lần lượt tại D và E (BC không là đuờng kính của đường tròn tâm O). Đường cao AH của tam giác ABC cắt DE tại K.
1) Chứng minh$\widehat{ADE}$=$\widehat{ACB}$
2) Chứng minh K là trung điểm DE
3) Trường hợp K là trung điểm của AH. Chứng minh rằng đường thẳng DE là tiếp tuyến chung ngoài của đường tròn đường kính BH và đường tròn đường kính CH

Bài viết đã được chỉnh sửa nội dung bởi thusang3605: 04-06-2012 - 15:25


#278
Doilandan

Doilandan

    Trung sĩ

  • Thành viên
  • 141 Bài viết
Bài 95 câu d)
Các bạn giải thích dùm mình tại sao góc MFA = góc MDA?

#279
Poseidont

Poseidont

    Dark Knight

  • Thành viên
  • 322 Bài viết
Bài 105: Cho (O) đường kính AB , M là điểm đối xứng với O qua A, đường thẳng d đi qua M cắt (O) tại C và D (C nằm giữa M và D), AD cắt BC tại I.CMR tam giác IOA cân
:lol: :namtay >:)

Nguyễn Đức Nghĩa tự hào là thành viên VMF


#280
pidollittle

pidollittle

    Trung sĩ

  • Thành viên
  • 132 Bài viết

Bài 104: Cho tam giác ABC vuông tại A. Một đường tròn (O) đi qua B và C cắt các cạnh AB, AC của tam giác ABC lần lượt tại D và E (BC không là đuờng kính của đường tròn tâm O). Đường cao AH của tam giác ABC cắt DE tại K.
1) Chứng minh$\widehat{ADE}$=$\widehat{ACB}$
2) Chứng minh K là trung điểm DE
3) Trường hợp K là trung điểm của AH. Chứng minh rằng đường thẳng DE là tiếp tuyến chung ngoài của đường tròn đường kính BH và đường tròn đường kính CH

bạn nào vẽ hình giúp mình nhé - thanks
a) $\widehat{ADE}=\widehat{BCA}$ ( vì tứ giác DECB nội tiếp )

b) có $\widehat{BAH}=\widehat{BCA}=\widehat{ADE}$
$\Rightarrow$$\Delta$DKA cân tại K
$\Rightarrow$ DK= KE
c) có DK=KE=AK=HK và $\widehat{ADE}=90^{o}$
$\Rightarrow$tứ giác DHEA là hình vuông
gọi I và F lần lượt là tâm đường tròn đường kính BH và đường tròn đường kính CH
$\Rightarrow$$\widehat{IDE}=\widehat{IDH}+\widehat{HDE} = \frac{1}{2}\widehat{BDA}=90^{o}$ (1)
Tượng tự ta có $\widehat{DEF}$= $90^{o}$ (2)
$\Delta BDH$ vuông tại D có DI là trung tuyến và $\Delta HEC$ vuông tại E có EF là trung tuyến
nên IF = IH + HF= DI + EF (3)
1,2,3 $\Rightarrow$ đpcm
p/s @thusang3605 hoàn thành nhiệm vụ . bạn cảm ơn mình đi :)




1 người đang xem chủ đề

0 thành viên, 1 khách, 0 thành viên ẩn danh